Practicing Success

Target Exam

CUET

Subject

Chemistry

Chapter

Inorganic: Coordination Compounds

Question:

Match Column I with Column II

Column I

(Complex)

Column II

(Magnetic moment)

(A) \(K[Cr(H_2O)_2(C_2O_4)_2].3H_20\) (p) 1.73 BM
(B) \(K_4[Mn(CN)_6]\) (q) 5.92 BM
(C) \([Co(NH_3)_5Cl]Cl_2\) (r) 3.87 BM
(D) \(Cs[FeCl_4]\) (s) Zero
Options:

A-r, B-p, C-s, D-q

A-p, B-r, C-q, D_s

A-q, B-r, C-p, D-s

A-s, B-r, C-q, D-p

Correct Answer:

A-r, B-p, C-s, D-q

Explanation:

The correct answer is option 1. A-r, B-p, C-s, D-q

A. The given complex is \(K[Cr(H_2O)_2(C_2O_4)_2].3H_20\)

Let the oxidation number of \(Cr\) be \(x\)

\(∴ 1 + x + 2 × 0 + 2(−2) = 0\)

\(⇒ x = 3\)

So, the electronic configuration of Cr in +3 oxidation state is

\(Cr^{3+} = _{18}Ar] 4s^0 3d^3\)

Here, the number of unpaired electrons, \(n = 3\)

Therefore, the magnetic moment is

\(\mu = \sqrt{3(3 + 2)} = 3.87 BM\) i.e., (r)

B. The given complex is \(K_4[Mn(CN)_6]\)

Let the oxidation number of \(Mn\) be \(x\)

\(∴ 4 + x + 6 × (−1) = 0\)

\(⇒ x = 2\)

So, the electronic configuration of Mn in +2 oxidation state is

\(Mn^{2+} = _{18}Ar] 4s^0 3d^5\)

 But due to the presence of a strong field ligand pairing of the electrons occurs and so the d-orbital can be presented as

Here, the number of unpaired electrons, \(n = 1\)

Therefore, the magnetic moment is

\(\mu = \sqrt{1(1 + 2)} = 1.73 BM\) i.e., (p)

C. The given complex is \([Co(NH_3)_5Cl]Cl_2\)

Let the oxidation number of \(Co\) be \(x\)

\(∴ x + 3 × 0 + (−1) + (−2)= 0\)

\(⇒ x = 3\)

So, the electronic configuration of Co in +3 oxidation state is

\(Co^{3+} = _{18}Ar] 4s^0 3d^6\)

But due to the presence of the strong field ligand pairing of electrons occurs in the d-orbital in the \(t_{2g}\) level

So, the number of unpaired electrons, \(n = 0\) [ due to pairing]

Therefore, the magnetic moment is

\(\mu = \sqrt{0(0 + 2)} = 0 BM\) i.e., (s)

D. The given complex is \(Cs[FeCl_4]\)

Let the oxidation number of \(Fe\) be \(x\)

\(∴ 1 + x + 4 × (−1) = 0\)

\(⇒ x = 3\)

So, the electronic configuration of Fe in +3 oxidation state is

\(Fe^{3+} = _{18}Ar] 4s^0 3d^5\)

Here, the number of unpaired electrons, \(n = 5\)

Therefore, the magnetic moment is

\(\mu = \sqrt{5(5 + 2)} = 5.92 BM\) i.e., (q)

The correct answer is (a) A-r, B-p, C-s, D-q